LSAT and Law School Admissions Forum

Get expert LSAT preparation and law school admissions advice from PowerScore Test Preparation.

 alee
  • Posts: 57
  • Joined: Mar 21, 2012
|
#4189
hi guys,

Regarding Q17 of Preptest 57, Section 2, I understood the conclusion to be:

Conclusion: Your magazine publishes an anthology of poems first printed in your magazine -> your magazine could depend less on donations

But why is option E correct? After some thinking, I've come to following rationale, what do you think?

The conclusion makes a predictive statement about what would happen if the other magazine published an anthology of poems *first printed in that magazine*, *based on what has happened with BWR's experience in releasing such an anthology*. For this argument to be strong, it must be the case that BWR's anthology and the proposed anthology for the other magazine *are sufficiently similar*. However, if option E is true, then BWR actually produces an anthology that is *different in a relevant way* to the one that is proposed for the other magazine, since it does not just consist of poems 'first printed in that magazine' but also other poems 'not published in the regular issues of that magazine'.

Could you please clarify how to reason this out? I'm still not feeling 100% about it. Incidentally, currently taking the Advance Reasoning course, which is great.

Cheers
 Ellali
  • Posts: 2
  • Joined: May 28, 2012
|
#4193
i think your reasoning iss 100% correct :-D
the author draws the conclusion basing on the samilarity, however, E point out that the two anthologies are actually different.
 Steve Stein
PowerScore Staff
  • PowerScore Staff
  • Posts: 1153
  • Joined: Apr 11, 2011
|
#4202
Hi Alee,

Thanks for your question--in that one, the author is basically saying "This anthology idea makes us money, and your magazine's poems are similar, so you could make money with this same idea."

The question asks for the choice that most weakens the author's conclusion, and answer choice E, as you said, provides a point of important distinction. If the Brick Wall's anthology has other selling points, those might be why it sells so well. This hurts the conclusion that the other magazine could rely on similar sales.

Let me know whether that makes sense--thanks!

~Steve
 Nina
  • Posts: 81
  • Joined: Sep 11, 2012
|
#5613
for the correct answer E, how should we understand it? Should it be understood as, if it "contains a number of poems by famous poets not published in the regular issues of the magazine," then The Brick Wall Review probably didn't rely on its publishing policy (that if a poem is printed in one of its regular issues, the magazine also has the right to reprint it, without monetary compensation, in its annual anthology) to make money?

Many thanks!
 Steve Stein
PowerScore Staff
  • PowerScore Staff
  • Posts: 1153
  • Joined: Apr 11, 2011
|
#5631
Hi Nina,

In that one, the fact that the magazine reserves the right to republish from its old issues doesn't keep it from having famous poets' work in its annual anthology.

And, if they do have these special guest stars included, then that weakens the patron's comparison--the two magazines have similar material, but with this choice it seems that the Brick Wall Review anthology's success could likely be attributable to the famous special guest poets.

I hope that's helpful--let me know.

Thanks!

~Steve
 Nina
  • Posts: 81
  • Joined: Sep 11, 2012
|
#5635
Thank you so much, Steve! It is of great of to me :)
 sdlee4
  • Posts: 5
  • Joined: Sep 27, 2012
|
#5777
So, I am having some trouble understanding how answer choice (E) is correct and weakens the argument. I read the explanation on the other post, but I still need further clarification please.

From my understanding, I believe the argument's conclusion is that "If your magazine also published an anthology of poems first printed in your magazine, you could depend less on donations"

And so, I couldn't make the connection on how answer choice (E) would weaken this conclusion and while keeping all the premises true. Also, if you could, could you explain why all the wrong answers are incorrect as well.

Thanks,
Sam
 Steve Stein
PowerScore Staff
  • PowerScore Staff
  • Posts: 1153
  • Joined: Apr 11, 2011
|
#5785
Hi,

Thanks for your question. In that one, the patron suggests that an anthology of poems would be a good idea for the "Poetry Journal" because it was a good move for the Brick Wall Review.

This argument is based on the premise that the two journals deal with similar poetry.

The correct answer choice (E) weakens the comparison, because if the Brick Wall Review Annual happens to have the advantage of a bunch of famous poets, then the comparison breaks down--in other words, given the fact that the Brick Wall Review Annual might have the famous poets to thank for its success, there is now less reason to believe that Poetry Journal would benefit from a simple annual anthology (without a bunch of special guest poets), even if it does deal with similar subject matter.


Was there an answer choice that you found particularly appealing? If so, how did you see it weakening the patron's argument?

Let me know--thanks!

~Steve
 sdlee4
  • Posts: 5
  • Joined: Sep 27, 2012
|
#5789
Ah okay, so are you saying that because the correct choice weakens one of the premises in the argument which in this case is "After all, most poems...in The Brick Wall Review", in effect also weakens the conclusion?

I thought that all weaken/strengthen questions had to assume that all premises must be true.

I chose answer choice (C) because I thought this choice directly weakened the conclusion of the argument, "So, if your magazine also,....less on donations" if that is the conclusion of the argument.

Thanks
 Steve Stein
PowerScore Staff
  • PowerScore Staff
  • Posts: 1153
  • Joined: Apr 11, 2011
|
#5791
Thanks for your response.

All of the premises are still true--the two magazines do indeed feature similar poetry, and the Brick Wall Review does have a successful annual.

The question that follows says "Which of the following, if true,..."

So, as we consider this answer choice, we have to take it as true and assess the effects on the argument from the stimulus.

So, it's still true that the two journals deal with similar subject matter, but when we add correct answer choice E into the mix, then the Brick Wall Review actually has an unfair advantage that wasn't previously discussed.

The problem with answer choice C is that the patron's argument concerns the Poetry Journal's ability to count on an Annual for increased revenues. Since Answer choice C does not weaken the patron's argument, it can be safely ruled out.

I hope that clears it up--let me know. Thanks!

~Steve

Get the most out of your LSAT Prep Plus subscription.

Analyze and track your performance with our Testing and Analytics Package.